Tous les devoirs de 2nde 
Tous les cours et exercices corrigés 
Télécharger le sujet du devoir 
Télécharger le corrigé du devoir 


Exercice 1 Déterminer l'ensemble de définition des fonctions
 
  1. $\displaystyle f:x\mapsto \frac{1}{x^2-16}
  2. $\displaystyle g:x\mapsto \frac{\sqrt{3x-6}}{(x+3)(2x-5)}
  3. $\displaystyle h:x\mapsto \sqrt{(x-3)(5-x)}

Exercice 2 Résoudre les inéquations :
$\dsp(I_1) :\ (x+3)(2x-5)\leq 0
 
$\dsp(I_2) :\  \frac{2}{2x-3}\geq 1
Exercice 3 On considère les fonctions $\displaystyle f:x\mapsto \frac{1}{2}x^2 et $\displaystyle g:x\mapsto \frac{1}{x+1}.
 
Le but de l'exercice est de comparer les positions des courbes $\mathcal{C}_f et $\mathcal{C}_g représentatives des fonctions $f et $g.
 
  1. Déterminer l'ensemble de définition des fonctions $f et $g.
     
  2. Montrer que, pour tout nombre $x réel, $x^3+x^2-2=(x-1)(x^2+2x+2).
     
  3. Montrer que pour tout nombre $x réel, $x^2+2x+2=(x+1)^2+1.
    En déduire le signe de l'expression $x^2+2x+2.
     
  4. A l'aide de ce qui précède, déterminer la position relative des courbes $\mathcal{C}_f et $\mathcal{C}_g.

Exercice 4 Monsieur Dupré, PDG d'une société fabriquant du mobilier urbain, s'intéresse au bénéfice réalisé par sa société.
Il fabrique et vend, par semaine, $x lots de mobilier.
Le coût unitaire de production, en euros, $f(x) (coût de production pour un lot de mobilier) s'exprime en fonction du nombre de lots $x par l'expression: $f(x)=x+72.
A ce coût unitaire s'ajoute des frais de fonctionnement de l'usine de production s'élevant à 3 952 euros par semaine, quelle que soit la quantité de lots produite.
 
  1. Chaque lot étant vendu 200 euros, montrer que le bénéfice réalisé pour $x lots produits et vendus est:
     B(x)=-x^2+128x-3952 = (x-52)(76-x)

    Déterminer alors le nombre de lots que doit produire et fabriquer la société pour être rentable (pour avoir un bénéfice positif …).
     
  2. Montrer que $B(x)=-(x-64)^2+144.
    Etudier alors les variations de $B sur $[0;64] et sur $[64;+\infty[. Dresser le tableau de variations de $B.
    Quel est le bénéfice maximal que peut espérer Monsieur Dupré ? Pour combien de lots fabriqués et vendus ?
     


Solution:



Exercice 1
  1. $\displaystyle f:x\mapsto \frac{1}{x^2-16}.   $x ne doit pas prendre de valeurs telles que $x^2-16=0, soit $x^2=16, et donc, $x=4 ou $x=-4. Ainsi, $\mathcal{D}_f=\R\setminus \la-4;4\ra.
  2. $\displaystyle g:x\mapsto \frac{\sqrt{3x-6}}{(x+3)(2x-5)}.   $x ne doit pas prendre de valeurs telles que $3x-6<0 et $(x+3)(2x-5)=0, soit $x<2 et $x=-3 et $x=\frac{5}{2}. Ainsi, $\mathcal{D}_g=[2;+\infty[\ \setminus \la\frac{5}{2}\ra.
  3. $\displaystyle h:x\mapsto \sqrt{(x-3)(5-x)}.  
    $x ne doit pas prendre de valeurs telles que $(x-3)(5-x)<0, et donc, d'après le tableau de signes,
    $\mathcal{D}_h=[3;5].
    \begin{tabular}[t]{|c|ccccccc|}\hline
$x$&$-\infty$ & & $3$ & & $5$ & & $+\infty$ \\\hline
$x-3$ & & $-$ & \mbox{$0\hspace{-0.67em}\mid$} & $+$ & $|$ & $+$ &\\\hline
$5-x$ & & $+$ & $|$ & $+$ & \mbox{$0\hspace{-0.67em}\mid$} & $-$ &\\\hline
$(x-3)(5-x)$ & & $-$ & \mbox{$0\hspace{-0.67em}\mid$} & $+$ & \mbox{$0\hspace{-0.67em}\mid$} & $-$ & \\\hline
\end{tabular}}



 
Exercice 2
$\dsp(I_1) :\ (x+3)(2x-5)\leq 0
 

D'après le tableau de signes, les solutions de $(I_1) sont $\displaystyle \mathcal{S}=[-3;\frac{5}{2}].

\begin{tabular}[t]{|c|ccccccc|}\hline
$x$&$-\infty$ & & $-3$ & & $\frac{5}{2}$ & & $+\infty$ \\\hline
$x+3$ & & $-$ & \mbox{$0\hspace{-0.67em}\mid$} & $+$ & $|$ & $+$ &\\\hline
$2x-5$ & & $-$ & $|$ & $-$ & \mbox{$0\hspace{-0.67em}\mid$} & $+$ &\\\hline
$(x-3)(5-x)$ & & $+$ & \mbox{$0\hspace{-0.67em}\mid$} & $-$ & \mbox{$0\hspace{-0.67em}\mid$} & $+$ & \\\hline
\end{tabular}


 
$\dsp(I_2) :\  \frac{2}{2x-3}\geq 1 \Longleftrightarrow
  \frac{-2x+5}{2x-3}\geq 0
 

D'après le tableau de signes, les solutions de $(I_2) sont
$\displaystyle \mathcal{S}=\Big]\frac{3}{2};\frac{5}{2}\Big].

\begin{tabular}[t]{|c|ccccccc|}\hline
$x$&$-\infty$ & & $\frac{3}{2}$ & & $\frac{5}{2}$ & & $+\infty$ \\\hline
$-2x+5$ & & $+$ & $|$ & $+$ & \mbox{$0\hspace{-0.67em}\mid$} & $-$ &\\\hline
$2x-3$ & & $-$ & \mbox{$0\hspace{-0.67em}\mid$} & $+$ & $|$ & $+$ &\\\hline
$\frac{-2x+5}{2x-3}$ & & $-$ & \mbox{$\hspace{0.1em}|\hspace{-0.67em}\mid$} & $+$ & \mbox{$0\hspace{-0.67em}\mid$} & $-$ & \\\hline
\end{tabular}


 
Exercice 3
  1. Dans l'expression de $f(x), $x peut prendre n'importe quelle valeur réelle: $\mathcal{D}_f=\R, tandis que pour $g(x), $x ne doit pas prendre de valeur telle que $x+1=0, soit $x=-1, et donc, $\mathcal{D}_g=\R\setminus\la-1\ra.
     
  2. Pour tout réel $x, $(x-1)(x^2+2x+2) = x^3+2x^2+2x -x^2-2x-2 = x^3+x^2-2.
     
  3. Pour tout réel $x, $(x+1)^2+1=x^2+2x+1+1=x^2+2x+2.
    Pour tout nombre réel $x, $(x+1)^2\geq 0, et donc $x^2+2x+2=(x+1)^2+1\geq 1 >0.
    Ainsi, $x^2+2x+2 est toujours strictement positif.
     
  4. Pour comparer les positions des courbes $\mathcal{C}_f et $\mathcal{C}_g, on étudie le signe de $f(x)-g(x):
    $\displaystyle f(x)-g(x)=\frac{1}{2}x^2-\frac{1}{x+1}=\frac{x^3+x^2-2}{2(x+1)}
    et donc, d'après la question 1), $\displaystyle f(x)-g(x)=\frac{(x-1)(x^2+2x+2)}{2(x+1)}.

    \begin{tabular}[t]{|c|ccccccc|}\hline
    $x$&$-\infty$ & & $-1$ & & $1$ & & $+\infty$ \\\hline
    $x-1$ & & $-$ & $|$ & $-$ & \mbox{$0\hspace{-0.67em}\mid$} & $+$ &\\\hline
    $x^2+2x+2$ & & $+$ & $|$ & $+$ & $|$ & $+$ &\\\hline
    $2(x+1)$ & & $-$ & \mbox{$0\hspace{-0.67em}\mid$} & $+$ & $|$ & $+$ &\\\hline
    $\frac{(x-1)(x^2+2x+2)}{2(x+1)}$& & $+$ & \mbox{$\hspace{0.1em}|\hspace{-0.67em}\mid$} & $-$ & \mbox{$0\hspace{-0.67em}\mid$} & $+$ & \\\hline
  \end{tabular}


    Ainsi, $\mathcal{C}_f est au-dessus de $\mathcal{C}_g lorsque $x\in]-\infty;-1[\cup[1;+\infty[, et au-dessous lorsque $x\in]-1;1].
    Les deux courbes se coupent en $x=1.

Exercice 4
  1. $x lots produits et vendus rapportent $200 x euros. La production de ces $x lots coûtent $x\times f(x)=x(x+72) euros plus 3952 euros. Ainsi, le bénéfice est $B(x)=200x - \Big( x(x+72)+3952\Big)=-x^2+128x-3952.
     
    Par ailleurs, $(x-52)(76-x)=76x-x^2+52x-52\tm76=-x^2+128x-3952=B(x). Ainsi, le bénéfice pour $x lots produits et vendus est $B(x)=-x^2+128x-3952 = (x-52)(76-x).

    \begin{tabular}[t]{|c|ccccccc|}\hline
    $x$&$-\infty$ & & $52$ & & $76$ & & $+\infty$ \\\hline
    $x-52$ & & $-$ & \mbox{$0\hspace{-0.67em}\mid$} & $+$ & $|$ & $+$ &\\\hline
    $76-x$ & & $+$ & $|$ & $+$ & \mbox{$0\hspace{-0.67em}\mid$} &  $-$ &\\\hline
    $B(x)$& & $-$ & \mbox{$0\hspace{-0.67em}\mid$} & $+$ & \mbox{$0\hspace{-0.67em}\mid$} & $-$ & \\\hline
  \end{tabular}


    La société est rentable lorsque le bénéfice est positif, soit donc lorsque le nombre de lots $x produits et vendus est compris entre $52 et $76 lots.
     
  2. $-(x-64)^2+144=-(x^2-128x+64^2)+144=-x^2+128x-3952=B(x). Ainsi, pour tout $x, $B(x)=-(x-64)^2+144.
     
    $\bullet Sur $[0;64]: Soit $a et $b deux nombres quelconques de $[0;64] tels que $0\leq a< b\leq 64,
     
    alors $-64\leq a-64<b-64\leq 0,
    donc, $64^2\geq (a-64)^2>(b-64)^2\geq 0,
    d'où, $-64^2\leq -(a-64)^2<(b-64)^2\leq 0,
    soit, $-64^2+144\leq f(a)<f(b)\leq 144
     
    donc, $f est croissante sur $[0;64].
     
    $\bullet Sur $[64;+\infty[: Soit $a et $b deux nombres quelconques de $[64;+\infty] tels que $64\leq a< b,
     
    alors $0\leq a-64<b-64,
    donc, $0\leq (a-64)^2<(b-64)^2,
    d'où, $0\geq -(a-64)^2>(b-64)^2,
    soit, $144\geq f(a)>f(b)
     
    donc, $f est décroissante sur $[64;+\infty[.
     


    \begin{tabular}{|c|ccccc|}\hline
    $x$    & $0$     & & $64$ & & $+\infty$ \\\hline
           & & & $144$ &&\\
    $B(x)$ &         & \Large{$\nearrow$}& &\Large{$\searrow$}& \\

           & $-3952$ & &&&\\\hline
  \end{tabular}


    Le bénéfice maximum que peut espérer M. Duspré est de $144 euros, pour $64 lots produits et vendus. (remarque: pour $x=64 lots la société est bien rentable, cf. question 1)).




Tous les devoirs de 2nde 
Tous les cours et exercices corrigés 
Télécharger le sujet du devoir 
Télécharger le corrigé du devoir